Q21

 
shehadehr
Thanks Received: 2
Vinny Gambini
Vinny Gambini
 
Posts: 10
Joined: July 09th, 2012
 
 
 

Q21

by shehadehr Mon Dec 17, 2012 7:50 pm

I cannot find where to post the "subject" for new topic!

Quick question, how is the answer B and not D? If, J is in 3 then J is also in 8, but this violates constraint 4...

Am I missing something?

my setup is like this:

_ _ _ _ _ _ _ _ _ _
1 6 2 7 3 8 4 9 5 10

I put O is 4 and O in 9, so then J's have to go in 5 and 10, and since M comes before O and J, and since k and L have to be split up, I use all this, hit the questions, eliminate A B C and E, so D is left by default. Seriously not seeing B as the right answer at all :evil:

Edit: Nevermind, my answer key was wrong. However, could someone take a look at my setup and let me know what they think? I feel like it worked well and I was fast. I think what helps is that you remember if the same letter has to always go in the given paired laps (ie, laps 1 & 6: if its stated that M is in 1, then you know M is in 6 too, also, keeping in mind the M--OJ contraint, as well as others).
User avatar
 
ManhattanPrepLSAT1
Thanks Received: 1909
Atticus Finch
Atticus Finch
 
Posts: 2851
Joined: October 07th, 2009
 
 
 

Re: Q21

by ManhattanPrepLSAT1 Tue Dec 18, 2012 6:13 pm

Hey Ray!

You've got it. If you put O in 4+9, then J will go in 5+10.

_ _ _ O J _ _ _ O J

At this point M, K, and L are a bit up in the air. But that's okay, lets hit the answers. We need to find one that could be true.

(A) must be false as J swims laps 5 and 10.
(B) must be false as J swims laps 5 and 10.
(C) must be false as J swims laps 5.
(D) could be true.
(E) must be false as J swims laps 5.

Btw, your setup looks good! Here's a hint though, did you try framing this one? You might find that since the OJ chunk is limited to only 3 places, you might be able to work through the questions more quickly.

Image
User avatar
 
Crogati
Thanks Received: 2
Jackie Chiles
Jackie Chiles
 
Posts: 32
Joined: January 12th, 2013
 
 
 

Re: Q21

by Crogati Thu Mar 07, 2013 9:35 pm

I am confused how (c) is the answer. Doesn't L in spot 3 force K in spot 2 thus violating the rule "no KL"?
 
sumukh09
Thanks Received: 139
Atticus Finch
Atticus Finch
 
Posts: 327
Joined: June 03rd, 2012
 
 
trophy
Most Thanked
trophy
First Responder
 

Re: Q21

by sumukh09 Fri Mar 08, 2013 4:15 pm

Crogati Wrote:I am confused how (c) is the answer. Doesn't L in spot 3 force K in spot 2 thus violating the rule "no KL"?


D is the answer for 21
User avatar
 
ManhattanPrepLSAT1
Thanks Received: 1909
Atticus Finch
Atticus Finch
 
Posts: 2851
Joined: October 07th, 2009
 
 
 

Re: Q21

by ManhattanPrepLSAT1 Sun Mar 10, 2013 8:09 pm

sumukh09 Wrote:D is the answer for 21

Thanks sumukh09! Also, for Q21 if L swims in lap 3, that does not force K into lap 2. One possibility is that it goes

K M L O J K M L O J

This can be seen in the middle frame in the middle of the 3 following frames based on placing the OJ chunk:

Image